UFL Estate Planning: Quizzes

Ace your homework & exams now with Quizwiz!

Do insurance policy proceeds apply to either the income tax or estate tax treatment of a life insurance?

Yes - benefits received under a periodic settlement option are partially subject to income tax.

Eric and Tawny gift $120,000 to an Irrevocable Life Insurance Trust with Crummey provisions. The trust has, as beneficiaries, their three children. A few weeks later, Eric dies in an auto accident. Tawny, with the assistance of her attorney and Financial Planner, is calculating Eric's gross estate. How much of the gift will be brought back into Eric's gross estate? The annual exclusion is $14,000 for 2014. Split gifts are available. The 5/5 lapse rule is in effect. a. $0 b. $21,000 c. $42,000 d. $102,000

a. $0 This was a cash gift, not a gift of life insurance. Therefore, none of the gift will be included in Eric's gross estate as the trust is irrevocable.

Which of the following statements about split-dollar life insurance and its uses is incorrect? a. Stock redemption plans can be funded by split-dollar life insurance. b. All policy values and benefits of a split-dollar life insurance policy are subject to the claims of company general creditors. c. The insurance premium of a split-dollar life insurance contract is generally divided between the employee and the employer. d. The insurance death benefit of a split-dollar life insurance policy is generally divided between the employee and the employer.

b. All policy values and benefits of a split-dollar life insurance policy are subject to the claims of company general creditors. Only the portion of benefit in the policy that is attributable to the actual contributions of the company are subject to the claims of company creditors.

Which of the following is an advantage of a revocable living trust? a. Reduction in federal estate taxes. b. Avoidance of probate. c. Removal of asset appreciation from the grantor's gross estate. d. Distribution of the trust assets according to the terms of the grantor's will.

b. Avoidance of probate Option a is incorrect because use of a revocable living trust does not reduce the grantor's federal estate taxes because the full fair market value of the trust assets are included in the grantor's gross estate. Option c is incorrect for the same reason. Option d is incorrect because the trust agreement, not the grantor's will, controls the distribution of the trust assets.

Which of the following is NOT a feature of a testamentary trust? a. Creation under a last will and testament. b. Shifts the income tax burden to a lower-bracket taxpayer. c. Results in the inclusion of assets in the gross estate. d. Does not avoid probate.

b. Shifts the income tax burden to a lower-bracket taxpayer. All of the other answers are features of a testamentary trust.

Which of the following describes joint and survivorship life insurance? 1. It is generally not includible in any insured's gross estate, if owned in an ILIT. 2. It can provide liquidity to pay estate taxes at the death of the second insured. 3. It pays a partial benefit at the death of the first to die (administrative and estate taxes) with the remainder paid in full at the second death. 4. Premiums are usually less expensive than for individual policies on each of the two insureds for the same face amount. a. 1 and 2 b. 2 and 4 c. 1, 2 and 4 d. 1, 2, 3, and 4

c.

Which statement(s) is/are true for Generation Skipping Transfer Tax (GSTT)? 1. Applies to transfers to persons who are two generations or more lower than the transferor. 2. There are no exceptions for GSTT 3. There's a $5,340,000 lifetime exemption in 2014 for GSTT 4. Transfers qualifying for gift tax annual exclusion are also excluded from GSTT a. 1 and 4 b. 2 and 3 c. 1, 3 and 4 d. 4 only

c. 1, 3 and 4

The Generation Skipping Transfer Tax (GSTT) has all the following characteristics, except: A. GST gifts to direct skips qualifying for the annual exclusion are not subject to the tax. B. Assets transferred to a trust that has a grandchild as the sole beneficiary may be subject to both gift and generation skipping transfer tax. C. If all the children of a trust are grandchildren (whose parents are living) of the grantor then the trust is subject to GSTT. D.

d

Which of the following statements is incorrect? a. When a decedent's taxable estate is less than the applicable estate tax credit equivalency, the estate is said to be overqualified. b. When too few assets pass to a decedent's surviving spouse, and as such the decedent's taxable estate is greater than the applicable estate tax credit equivalency, the decedent's estate is said to be underqualified. c. An ABC Trust arrangement utilizes a General Power of Appointment Trust, a QTIP Trust, and a Bypass Trust to maximize the use of a decedent's applicable estate tax credit. d. The remainder beneficiary of a QTIP Trust is chosen by the surviving spouse.

d. Option d is incorrect because the ultimate beneficiary of a QTIP Trust is chosen by the grantor of the QTIP Trust. All of the other statements are correct.

Under which of the following circumstances would a decedent be considered to have died intestate? a. The decedent handwrote a will, but did not sign or date it. b. The decedent was not of "sound mind" when he signed his statutory will. c. The decedent failed to prepare a last will and testament. d. All of the above.

d. All of the above. Option a describes an invalid holographic will. Option b describes a situation in which the testator is not "of sound mind" and therefore cannot make a valid will. If the decedent dies without a valid will, he is said to have died intestate.

Prairie Dog Corporation (PDC), an oil drilling company, has a "key-person" variable universal life policy on Digger Phelps, its vice-president of drilling operations. The owner and beneficiary of the policy are the corporation. Which of the following is correct? a. Premiums paid by PDC are taxable income to Digger. b. Premiums paid by PDC are considered gifts to Digger. c. Premiums paid by PDC are tax deductible as a business expenses. d. Any death benefit paid will be nontaxable to PDC.

d. Any death benefit paid will be nontaxable to PDC. PDC is the owner and beneficiary of the policy. For the same reason, premiums are NOT considered a gift or taxable to Digger, nor will they appear in his gross estate. "Key person" life premiums are not deductible as a business expense. Any death benefit pad will be nontaxable to PDC.

Charlotte is getting ready for her first meeting with her new financial planner, Samantha. What information does Charlotte not need to bring to this meeting? a. Previously filed income tax and gift tax returns. b. A copy of her current will. c. A detailed list of Charlotte's assets and liabilities. d. Charlotte should bring all of the above information to her first meeting with Samantha.

d. Charlotte should bring all of the above information to her first meeting with Samantha.

Cheryl, age 58, is the owner of a closely-held partnership business which makes up 65% of her adjusted gross estate. More than half the assets of the corporation are real estate holdings. Cheryl wants to undertake a transfer of some sort to her son, Roger, to reduce her potential income tax obligations and possible future estate tax liability. Such a transfer would accomplish both of these goals and reduce Cheryl's interest in the business by 35%, meaning the business would make up only 30% of her adjusted gross estate. Cheryl will also be bequeathing $50,000 to her favorite public charity and the balance to her husband upon her death. In light of these activities and transfers, which of the following elections does Cheryl lose? a. Cheryl can no longer use the special use election. b. Cheryl can no longer use the reverse QTIP election. c. Cheryl can no longer use the Section 303 election. d. Cheryl gives up the right to use the 6166 election.

d. Cheryl gives up the right to use the 6166 election. The amount required to use the Section 6166 is that the ownership asset must make up at a minimum of 35% of the estate. Section 303 is not appropriate because this is a partnership and there is not stock in a partnership. Special use is for valuation of real property used in a trade or business. The reverse QTIP is a generation transfer tax election.

Elizabeth, a widow, has decided to set up trusts for each of her four grandchildren to take advantage of the generation skipping transfer tax exemption. In the current year, she gives each grandchild $280,000. If Elizabeth has not made any previous taxable gifts, on what amount will she owe gift tax? a. $1,064,000 b. $280,000 c. $266,000 d. None

d. None The $5.34 million GSTT exemption will fully cover her $1,064,000.

Of the following, which is not an issue when considering whether to deduct the adjusted basis or the fair market value of property contributed to a charitable organization? a. The current market rate of interest. b. The donor's current and projected adjusted gross income for the 5 years after the contribution. c. The fair market value of the donated property. d. The capital gains rate in effect at the time of the transfer.

d. The capital gains rate in effect at the time of the transfer. Option d is not an issue when deciding whether to deduct the adjusted basis or the fair market value since the transfer generally does not created a capital gain. All of the other options are issues to consider.

Kathi and Darrin, who are married, own their home together as community property. They purchased the home 17 years ago for $100,000. After many improvements and a surge in the market, the home is now worth $200,000. If Darrin died today and left his share of the home to his daughter Elizabeth, what is Kathi's basis in the home? $50,000 $100,000 $150,000 $200,000

$100,000 Kathi's one-half interest in the home will have a basis of $100,000 due to a step-to fair market value of both halves at Darrin's death because the property is owned as community property.

Lois, an elderly, single woman, recently came to you to, an estate planning professional, to discuss her estate plan. After a lengthy discussion you determine that Lois completed several transactions last year that may be subject to gift tax. The transactions you uncovered include: 1) Lois had a bank account in the amount of $15,000 that was owned fee simple. She wanted to make sure her son, Ronnie, could access the money "just in case" so she changed the ownership of the account to JTWROS in her and Ronnie's name equally. Ronnie has not made any withdrawals. 2) Feeling guilty about retitling her checking account JTWROS with her son, Lois decided to change the titling of her vintage automobile as JTWROS with her daughter, Joyce. Lois purchased the property for $15,000 and the fair market value of the property on the date of retitling was $30,000. Due to a high demand for this particular vintage model the value of the car today is $40,000. 3) Lois received a beneficiary designation in the mail for her $1,000,000 life insurance policy. The policy never had a beneficiary, so she designated her son, Ronnie and daughter, Joyce, as joint beneficiaries. Lois's basis in the policy is $200,000. 4) Lois has two stock portfolio accounts with a local brokerage firm valued at $200,000. Upon her advisor's suggestion, she retitled the account as a Transfer on Death account to "save taxes." Upon her death, the assets will transfer equally to her son, Ronnie, and her daughter, Joyce. 5) Lois's daughter Joyce has always been a little poor with budgeting her money. So it was no surprise to Lois that Joyce couldn't afford her daughter Katelyn's braces. Feeling sorry for Katelyn, Lois gave Joyce $50,000 for the braces. Lois later found out that the braces only cost $15,000 and Joyce spent the remaining money on elective cosmetic surgery. 6) Lois was beginning to become very concerned because her son Ronnie had never married. She was so happy he finally got married she gave Ronnie and his new wife, Sam, $40,000 so they could take a two month trip to Australia. When you inform her that you are concerned about some of these transactions and that she may need to file a gift tax return she states, "you obviously must not be a very good planner because none of my other planners ever told me that, besides it would be ridiculous for me to pay tax on things I want to give to my family that I purchased with my hard earned money that was already taxed." After more discussion, Lois confesses to you that you are highly recommended and frankly, she has already used almost every planner in town and since they have all declined to represent her, she is confident that you will do the right things. Calculate the value of Lois's total qualified transfers for the current year. $0 $15,000 $35,000 $50,000

$0 Lois did not have any qualified transfers. A qualified transfer is a transfer directly to a medical provider for qualified expenses or to a qualified educational institution for tuition. While $50,000 was used for medical expenses, the money was not paid directly to the medical provider. In addition, the elective cosmetic surgery is not a qualified medical expense.

Property disclaimed by a surviving spouse will still qualify for the unlimited marital deduction. True False

False. If the surviving spouse properly disclaims certain property, that property will not qualify for the unlimited marital deduction.

Overqualification means that the decedent used too much of his applicable estate tax credit. True False

False. Overqualification occurs when too many assets pass to the surviving spouse and the decedent fails to take advantage of his applicable estate tax credit.

Which of the following assets would pass through probate? A life insurance policy with a named beneficiary. Assets held in trust. A pay-on-death account with a named beneficiary. Household goods.

Household goods All of the other options describe assets that do not pass through probate.

One advantage of the unlimited marital deduction is that the payment of any estate taxes can be deferred until the death of the surviving spouse. True False

True. This is an advantage of the unlimited marital deduction.

Curtis, who was married, recently died owning several assets. Given the assets below, determine whether each asset should be included in Curtis' probate estate. Curtis's portion of acreage owned as community property with his wife. Yes, this item is included in the probate estate. No, this item is not included in the probate estate.

Yes, this item is included in the probate estate. Community property is always included in the gross estate

In which of the following situations would the use of a QDOT be appropriate? a. Tom dies and is survived by his wife, Tina, who is not a U.S. citizen. b. Regina dies and is survived by her husband, Raul, who becomes a U.S. citizen two months after Regina's death. c. Harold dies and does not have a surviving spouse but has a significant other. d. Franz, who is not a U.S. citizen, dies and is survived by his wife, Francine, who is a U.S. citizen.

a. Option b does not describe a situation in which the use of a QDOT would be appropriate because Raul became a U.S. citizen prior to the due date of the estate tax return and therefore, any property transfers to Raul would qualify for the unlimited marital deduction. Option c is not correct because there is no reason to use a QDOT if Harold does not have a surviving spouse. Option d is not correct because a QDOT is used when the surviving spouse is not a U.S. citizen.

Amanda has been married to Javier for 25 years. Javier is a Honduran citizen. Amanda would like to make an inter vivos transfer to Javier. What is the maximum amount that Amanda can transfer to Javier without incurring transfer taxes or utilizing any of her applicable credit during 2014? a. $145,000 b. $1,000,000 c. $5,250,000 d. Unlimited

a. $145,000 There is a special annual exclusion for noncitizen spouses of $145,000. A spouse can transfer up to $145,000 to his noncitizen spouse without incurring gift taxes.

Eugene is considering having his attorney prepare a springing power of attorney in which his gives his friend, Eleanor, the power to handle his finances. Why should Eugene include such a document in his overall estate plan? a. In the event that Eugene becomes disabled, Eleanor will be able to pay Eugene's bills. b. Eleanor is not legally competent. c. Eleanor is only 16 years old. d. Eugene wants Eleanor to be able to handle all of his finances immediately.

a. In the event that Eugene becomes disabled, Eleanor will be able to pay Eugene's bills. Eugene should not make Eleanor the agent of his springing power of attorney if she is not legally competent or is not of the age of majority. If Eugene wants Eleanor to be able to handle his finances immediately, he should not use a spring power of attorney, which only becomes effective upon the principal's disability or incapacity.

Estate planning is the process of accumulation, management, conservation, and transfer of wealth considering legal, tax, and personal objectives. a. True b. False

a. True This is the definition of estate planning.

Which of the following accurately describes a life estate? a) An interest in property for a specified number of years. b) An interest in property that ceases upon the death of the owner of the life estate. c) An undivided interest in property held by two or more related or unrelated persons. d) A complete interest in property with all the rights associated with outright ownership.

b) A complete interest in property with all the rights associated with outright ownership.

Rachel died eight months ago and her executor is finalizing her estate tax return. The executor has determined that Rachel's adjusted gross estate is $10,120,000 and that her estate is entitled to a charitable deduction in the amount of $500,000. Calculate the estate tax liability for Rachel's estate. a. $336,800 b. $1,575,000 c. $1,712,000 d. $3,793,800

c. $1,712,000 Subtract the charitable deduction from the adjusted gross estate to get the taxable estate ($10,120,000 - $500,000 = $9,620,000). The tentative tax on the taxable estate is $3,793,800 ($345,800 + 0.40 ($8,620,000)). Subtract the applicable estate tax credit to determine the federal estate tax liability ($3,793,800 - $2,081,800 = $1,712,000).

This year, Dottie donated $10,000 in cash to her church and she also donated medical supplies with a fair market value and adjusted basis of $20,000 to the Red Cross. Dottie's AGI for this year is $50,000. What is Dottie's charitable income tax contribution deduction for the year? a. $10,000 b. $20,000 c. $25,000 d. $30,000

c. $25,000 The deduction of charitable donations in the form of cash is limited to 50% of AGI. Dottie's AGI is $50,000, so the deduction of any cash donations to a public charity will be limited to $25,000. The deduction of charitable donations of ordinary income property is limited to the lesser of the adjusted basis or the fair market value of the property. Dottie has made a total donation of $30,000 this year, but her deduction will be limited to $25,000.

Which of the following transfers would result in gift tax? a) Bob gifts $11,000 to his daughter Barbie. b) Elroy gifts $50,000 to his wife, Elizabeth, who is a U.S. citizen. c) Adam gives his favorite employee, Aaron, a new car at Aaron's retirement worth $20,000. d) Pete transfers $20,000 to his ex-wife, Patricia. Pete and Patricia were divorced five years ago.

d) Pete transfers $20,000 to his ex-wife, Patricia. Pete and Patricia were divorced five years ago. Option a would not result in gift tax because the gift does not exceed the annual exclusion. Option b is incorrect because a person can gift an unlimited amount to his or her spouse without incurring gift tax. Option c is incorrect because transfers in a business setting are presumed to be compensation. If Pete had transferred $20,000 to Patricia pursuant to a divorce decree, there would be no taxable gift, but transfers to an ex-spouse five years after the divorce was final are not considered "transfers pursuant to a divorce decree."

Which of the following statements is incorrect? a. When a decedent's taxable estate is less than the applicable estate tax credit equivalency because of the overuse of the marital deduction, the estate is said to be overqualified. b. When too few assets pass to a decedent's surviving spouse, and as such the decedent's taxable estate is greater than the applicable estate tax credit equivalency, the decedent's estate is said to be underqualified. b. An ABC Trust arrangement utilizes a General Power of Appointment Trust, a QTIP Trust. and a Bypass Trust to maximize the use of a decedent's applicable estate tax credit. d. The ultimate beneficiary of a QTIP Trust is selected by the surviving spouse.

d. The ultimate beneficiary of a QTIP Trust is selected by the surviving spouse. Option d is incorrect because the ultimate beneficiary of a QTIP Trust is chosen by the grantor of the QTIP Trust. All of the other statements are correct.

Which one of the following transfers made this year by 85-year old Jennifer is not sooner or later subject to the Generation Skipping Transfer Tax? A. A gift of a remainder interest in a trust just established which is paying an income interest to Jennifer. The remainderman is a grandson whose parents died in an auto accident earlier this year before the inception of the trust. B. A transfer by Jennifer of $15,000 to a UTMA account established by her son for Jennifer's granddaughter. C. An irrevocable trust which pays income to Jennifer for 10 years and then pays the remainder to her grandniece who is only 25 years younger than Jennifer. D. An irrevocable trust which pays income to Jennifer's daughter for life, then distributes the remainder to the grandchild of a friend and that grandchild is currently31 years old.

A. A gift of a remainder interest in a trust just established which is paying an income interest to Jennifer. The remainderman is a grandson whose parents died in an auto accident earlier this year before the inception of the trust.

Provided that a survivorship clause does not require the surviving spouse to survive the decedent for more than nine months, any transfers subject to the survivorship clause will qualify for the unlimited marital deduction. True False

False. In order to qualify for the unlimited marital deduction, the survivorship clause may not exceed six months and the surviving spouse must actually survive the survivorship period.

Karen, age 58, recently visited her attorney to discuss the appropriate estate planning documents she needed to effectuate her estate planning goals. Given the following goals, which document is appropriate to clarify Karen's burial wishes? Do Not Resuscitate order. Last Will and Testament. Living Will Power of Appointment Power of Attourney for Health Care Power of Attorney Side Instructional Letter

Side Instructional Letter The side instructional letter is the appropriate place to identify Karen's burial wishes.

Which of the following is not a method for transferring property outside of the probate process? State Contract Law State Intestacy Law State property titling law with survivorship feature State Trust Law

State Intestacy Law Property transferred via the state intestacy law will pass through probate.

Paula, a single woman, transferred $2,000,000 to a GRAT naming her two sons as the remainder beneficiaries, while retaining an annuity with a present value of $860,000. If this is the only transfer that Paula made during the year, what is Paula's total taxable gift for the year? a. $1,112,000 b. $1,140,000 c. $1,972,000 d. $2,000,000

b. $1,140,000 The present value of the expected future remainder interest is a gift of a future interest subject to gift tax. The value of the expected future remainder interest is $1,140,000 ($2,000,000 - $860,000). Because this is a gift of a future interest, it does not qualify for the annual exclusion.

Which of the following is not necessary to properly execute a Section 303 stock redemption? a. The value of the stock must be greater than 35% of the decedent's adjusted gross estate, including gifts made in the last 3 years. b. The 303 redemption can only be used if the corporation has the cash to redeem the shares. c. The 303 redemption can be made without a positive earnings and profits account. d. The Section 303 redemption is limited to an amount that cannot exceed the death taxes of the estate, plus funeral and administrative expenses for which the decedent is liable.

c. The 303 redemption can be made without a positive earnings and profits account. The closely-held stock must make up 35% of the decedent's adjusted gross estate value and must be the stock of a closely-held firm. The E and P account must be positive or there is no need for a 303 redemption.

Jose recently died with a probate estate of $900,000. He was predeceased by his wife, Guadalupe, and his daughter, Lucy. He has two surviving children, Pete and Fred. Jose was also survived by eight grandchildren, Pete's three children, Naomi, Daniel, Nick; Fred's three children, Heather, Chris and Steve; and Lucy's two children, David and Rachel. Jose's will states the following "I leave everything to my three children. If any of my children shall predecease me then I leave their share to their heirs, per stirpes." Which of the following statements is correct? a. Under Jose's will David will receive $225,000. b. Under Jose's will Chris will receive $150,000. c. Under Jose's will Nick will receive $100,000. d. Under Jose's will Fred will receive $300,000.

d. Under Jose's will Fred will receive $300,000. Under the will Pete and Fred will each receive 1/3 shares. Lucy's 1/3 share will flow to her children, with each of them receiving 1/2 of the 1/3 share.

Nate owns the following property: A personal residence titled fee simple valued at $500,000. A $500,000 life insurance policy on his own life. The only named beneficiary is Nate's brother Jaime, who died 6 months ago leaving two children, Michael and Kristi. A car valued at $15,000 titled JTRWROS with Nate's mother. An IRA valued at $400,000 with Nate's mother as the named beneficiary. What is the current value of Nate's probate estate? $500,000 $1,000,000 $1,400,000 $1,415,000

$1,000,000 The probate estate will include the personal residence and the life insurance policy. The life insurance policy is included because the named beneficiary was already dead at Nate's death. The car is not included because of the JTWROS ownership, thus it transfers by operation of law. The IRA is not included because there is a living named beneficiary and thus will transfer via contract law.

Lois, an elderly, single woman, recently came to you to, an estate planning professional, to discuss her estate plan. After a lengthy discussion you determine that Lois completed several transactions last year that may be subject to gift tax. The transactions you uncovered include: 1) Lois had a bank account in the amount of $15,000 that was owned fee simple. She wanted to make sure her son, Ronnie, could access the money "just in case" so she changed the ownership of the account to JTWROS in her and Ronnie's name equally. Ronnie has not made any withdrawals. 2) Feeling guilty about retitling her checking account JTWROS with her son, Lois decided to change the titling of her vintage automobile as JTWROS with her daughter, Joyce. Lois purchased the property for $15,000 and the fair market value of the property on the date of retitling was $30,000. Due to a high demand for this particular vintage model the value of the car today is $40,000. 3) Lois received a beneficiary designation in the mail for her $1,000,000 life insurance policy. The policy never had a beneficiary, so she designated her son, Ronnie and daughter, Joyce, as joint beneficiaries. Lois's basis in the policy is $200,000. 4) Lois has two stock portfolio accounts with a local brokerage firm valued at $200,000. Upon her advisor's suggestion, she retitled the account as a Transfer on Death account to "save taxes." Upon her death, the assets will transfer equally to her son, Ronnie, and her daughter, Joyce. 5) Lois's daughter Joyce has always been a little poor with budgeting her money. So it was no surprise to Lois that Joyce couldn't afford her daughter Katelyn's braces. Feeling sorry for Katelyn, Lois gave Joyce $50,000 for the braces. Lois later found out that the braces only cost $15,000 and Joyce spent the remaining money on elective cosmetic surgery. 6) Lois was beginning to become very concerned because her son Ronnie had never married. She was so happy he finally got married she gave Ronnie and his new wife, Sam, $40,000 so they could take a two month trip to Australia. When you inform her that you are concerned about some of these transactions and that she may need to file a gift tax return she states, "you obviously must not be a very good planner because none of my other planners ever told me that, besides it would be ridiculous for me to pay tax on things I want to give to my family that I purchased with my hard earned money that was already taxed." After more discussion, Lois confesses to you that you are highly recommended and frankly, she has already used almost every planner in town and since they have all declined to represent her, she is confident that you will do the right things. Calculate Lois's taxable gifts for the current year. $40,000 $55,000 $63,000 $105,000

$63,000 The retitling of the bank account is not a complete gift. It will not be complete until Ronnie actually withdrawals from the account. The retitling of the vintage automobile is a completed gift of one-half of the value. The beneficiary designation is not a gift. A Totten trust, or a Transfer on Death account, is a form of beneficiary designation and is not a completed gift. Taxable gifts is a term of art meaning gross gifts minus annual exclusions, which in this case are three (Joyce, Ronnie, and Sam), totaling $42,000. 1. Retitling of Bank Account $0 2. Retitling of Vintage Automobile $15,000 3. Change of beneficiary designation $0 4. Retitling of Stock accounts $0 5. Transfer of cash to Joyce for braces $50,000 6. Transfer of cash to Ronnie for honeymoon $40,000 Total Gifts Made $105,000 Less Annual Exclusion (3 People x $14,000) ($42,000) Total Taxable Gifts $63,000

Brody and Tanya recently sold some land they owned for $150,000. They received the land five years ago as a wedding gift from Brody's Aunt Jeanette. Aunt Jeanette purchased the land many years ago when the property was worth $20,000. At the date of the gift, the property was worth $100,000 and Aunt Jeanette paid $47,000 in gift tax. What is the long term capital gain on the sale of the property? $42,400 $50,000 $92,400 $130,000

$92,400 In general, when a donor makes a gift of property other than cash to a donee, the donee will take the property at the donor's adjusted basis. The holding period of the donee will include the holding period of the donor for purposes of subsequent transfers and the determination of long or short-term capital gains. An exception to the general basis rule occurs when the donor gives property with a fair market value in excess of his adjusted basis and the donor pays gift tax. The gift tax associated with the appreciation is added to the donor's original adjusted basis to determine the donee's basis. $20,000 + ($47,000 X $80,000/$100,000) = $57, 600. $150,000 - $57,600 = $92,400

Under what circumstances would property be subject to ancillary probate? a) If the decedent is a resident of one state and owns real property in another state. b) If the decedent is a tenant in common with an unrelated person. c) If the decedent was a resident of a community property state. d) If the decedent owns a life estate in real property located in a state other than his state of residence.

(a) If the decedent was a resident of a community property state.

You are a financial planner and you are preparing for a meeting with your new client, Anne. What would you be most likely to ask Anne to bring to the meeting with her? a. Pictures of her children b. Her parents c. Any will d. Sales records for her ex-husband's business

C. Any will You would be most likely to ask Anne to bring any will with her. In addition, you would be likely to request copies of any other estate planning documents as well as tax documents.

Big Mike, a very generous man, has given his granddaughter, Jordan, a gift equal to $5.34 million last year and paid any relevant taxes. He now wants to give his grandson, Colin, a gift of $5.34 million plus the annual exclusion of $14,000 on his birthday and wants to know what his total outflow will be for the gifts and any other related taxes. The amount of his cash flows related to the gift to Colin is? A. $5.354 million B. $7.476 million C. $9.626 million D. $10.4804 million

D. $10.4804 million

Curtis, who was married, recently died owning several assets. Given the assets below, determine whether each asset should be included in Curtis' probate estate A $500,000 life insurance policy on his own life. His daughter Ann was the named beneficiary and received the proceeds 40 days after Curtis' death. Yes, this item is included in the probate estate. No, this item is not included in the probate estate.

No, this item is not included in the probate estate. The life insurance policy had a designated beneficiary who was alive at Curtis's death because she received the property, thus the proceeds are not included in Curtis's probate estate

Brett died recently leaving all his assets in a trust for his wife Greer. Brett was concerned that Greer would not be able to manage her money adequately to maintain her standard of living for the rest of her life. Therefore, he placed the assets into a spendthrift trust and gave Greer the right to receive a certain amount of income each year. Brett appointed his good friend Paul to be the trustee of the trust. How is Paul's ownership classified? Paul holds a life estate over the property. Paul holds the legal title to the property. Paul holds the equitable title to the property. Paul does not hold an interest in the property.

Paul hold the legal title to the property. Paul holds the legal title to the property as trustee for the trust. Greer as the beneficiary holds the equitable title. A life estate identifies the person who has a current beneficial right in the property, which in this case would be Greer.

Rosie and her brother Michael decided recently to purchase an RV together. They both want to use the RV to take their families camping. The price for the RV was $10,000. Since Michael expects to use the RV 60% of the time and Rosie 40% of the time, Michael contributed $6,000 and Rosie contributed $4,000. Their ownership percentage equals their contribution percentage. Which type of property titling must the RV be to reflect their ownership interest? Fee Simple JTWROS Tenancy in Common Tenancy by the Entirety Community Property

Tenancy in Common Fee Simple ownership is for one owner. They cannot own the property JTWROS because they own unequal ownership percentages. Tenancy by the Entirety and Community Property must be owned between married people.

Which of the following is not a common estate planning goal? a. Maximizing transfer costs. b. Minimizing transfer taxes. c. Providing for liquidity at death. d. Fulfilling client's healthcare decisions.

a. Maximizing transfer costs. Minimizing transfer costs, not maximizing transfer costs, is a common estate planning goal. All of the other answers are common estate planning goals.

Elizabeth, who is not a licensed attorney, recently started her own financial planning practice. Which of the following activities would be considered the unauthorized practice of law? a. Preparing a last will and testament for her first client. b. Helping clients to identify their financial planning goals. c. Preparing financial statements for prospective clients. d. Referring clients to her brother, Jack, who happens to be a licensed attorney.

a. Preparing a last will and testament for her first client. Only licensed attorneys should prepare last will and testaments for clients.

Of the following, which property transfers at death by contract? a. Roth IRAs. b. Property titled Joint Tenancy with Rights of Survivorship (JTWROS). c. An Irrevocable Living Trust. d. A Grantor Retained Annuity Trust (GRAT).

a. Roth IRAs. Only the Roth IRA transfers property at death by contract. The beneficiary designation is the contract, and at the death of the account owner, the account assets will be transferred to the beneficiary. All of the others transfer by state property titling law or by state trust law.

Which of the following applies to the income tax or estate tax treatment of life insurance policy proceeds? a. Benefits received under a periodic settlement option are partially subject to income tax. b. Death proceeds are includible in the gross estate of the decedent if the decedent was the insured regardless of ownership. c. Payments under a cashout settlement option are partially subject to income tax. d. For a personally owned life insurance policy premiums are deductible if made as part of a court ordered child or spousal support plan (QDRO).

a. Periodic annuity settlement benefits are not fully subject to income tax because the recipient has a tax basis equal to the original proceeds. Proceeds are includible in estate for tax purposes only if grantor retained an incident of ownership. Life insurance premiums are not deductible when personally owned, and the 3-year rule applies to a life insurance policy regardless of irrevocability.

Curtis, who was married, recently died owning several assets. Given the assets below, determine whether each asset should be included in Curtis' probate estate. His personal residence worth $250,000 titled fee simple. Yes, this item is included in the probate estate. No, this item is not included in the probate estate.

Yes, this item is included in the probate estate. Items owned fee simple is always included in the gross estate

Chad and Ross (both males) have been involved in an intimate relationship for the past 25 years. Chad's family is quite wealthy, and has provided Chad with every "extra" in life. Unfortunately, Chad's family is also very conservative and they do not approve of Chad's relationship with Ross. Chad was diagnosed with cancer last year and given only 12-15 months to live. Chad plans to leave the substantial wealth he has inherited over the years to Ross. After a few too many glasses of wine last Christmas, Chad's mother proclaimed, "Chad, I hope you have a great estate planning attorney, because I will spend every penny I have to keep Ross from inheriting a dime from you!" In a fit of rage, Chad has come to you, an estate planning attorney, and asks you to recommend ways he can ensure that Ross will receive his assets. Which of the following would you be least likely to recommend to Chad to meet his objectives? a) A well-drafted will leaving everything to Ross with a no-contest clause. b) A revocable living trust created and funded now with Ross as the beneficiary at Chad's death. c) An irrevocable trust created and funded with Chad as the income beneficiary and Ross as the remainder beneficiary. d) Retitling all assets as JTWROS.

a) A well-drafted will leaving everything to Ross with a no-contest clause. While all of these options may seem to accomplish Chad's goal, option a has the most inherent risk. The trust options and titling option are much less likely to be susceptible to fraud and undue influence claims. The use of a will in this situation is very susceptible to a contest. The no-contest clause is irrelevant because Chad did not leave anything to anyone else to encourage them not to contest.

Trey decides to set up a trust for the benefit of his two sons, Ronnie and Chad. Trey makes an annual contribution to the trust in the amount of $28,000 and gives each son the right to withdraw up to $14,000. In the current year, when the total trust assets are $52,000, Ronnie decides to withdraw $14,000, but Chad does not withdraw anything. What is the result of Chad's decision not to withdraw any of Trey's contribution to the trust? a) Chad has made a taxable gift to Ronnie of $4,500. b) Ronnie has made a taxable gift to Chad of $14,000. c) Trey has made a taxable gift to Ronnie of $14,000. d) All of the above.

a) Chad has made a taxable gift to Ronnie of $4,500. This question addresses the 5/5 Lapse Rule. The 5/5 Lapse Rule states that a taxable gift has been made where a power to withdraw in excess of $5,000 or 5% of the trust assets is lapsed by the powerholder. In this case, Chad has allowed his power to withdraw $14,000 to lapse. As a result, Chad has made a gift to himself of $4,500 ($7,000-($5,000/2)) and a gift to Ronnie of $4,500 ($7,000-($5,000/2)).

Which of the following is true concerning the 5/5 Lapse Rule? a) The 5/5 Lapse Rule deems that a taxable gift has been made where a power to withdraw in excess of $5,000 or five percent of the trust assets is lapsed by the powerholder. b) The 5/5 Lapse Rule only comes into play with a single beneficiary trust. c) Amounts that lapse under the 5/5 Lapse Rule qualify for the annual exclusion. d) Gifts under the 5/5 Lapse Rule do not have to be disclosed on a gift tax return.

a) The 5/5 Lapse Rule deems that a taxable gift has been made where a power to withdraw in excess of $5,000 or five percent of the trust assets is lapsed by the powerholder. Option a is the definition of the 5/5 Lapse Rule. Option b is incorrect because the 5/5 Lapse Rule does not come into play with a single beneficiary trust because a person cannot make a taxable gift to himself. Option c is incorrect because amounts that lapse under the 5/5 Lapse Rule do not qualify for the annual exclusion. Option d is incorrect because gifts under the 5/5 Lapse Rule do have to be disclosed on a gift tax return.

Anne recently died. Anne is survived by her husband, Edward, and daughter, Catherine. Which of the following would be a qualifying property transfer for the purposes of the unlimited marital deduction? a. Anne leaves ownership of certain copyrights to Edward. b. Property transferred to a credit shelter trust for the benefit of Catherine, with Edward as the trustee. c. Anne leaves her beach house to Edward, subject to the condition that if Edward does not survive Anne's sister, Anne's sister will get the property. d. The $1,000,000 life insurance policy on Anne's life owned by Edward.

a. Although copyrights are terminable interests, no person other than Edward has any interest in the property, since all rights were given to Edward. Therefore, the transfer of the copyrights to Edward will qualify for the marital deduction. Option b does not qualify for the unlimited marital deduction because even though Edward is trustee, and has legal title to the property inside the trust, he does not have beneficial title to the property. Option c does not qualify for the unlimited marital deduction because the transfer to Edward is a terminable interest. Option d does not qualify for the unlimited marital deduction because the proceeds of a life insurance policy owned by Edward on Anne's life will not be included in Anne's gross estate.

Natalie and her younger sister Kate purchased a beach-front condominium together 15 years ago. They own the property as a joint tenancy with rights of survivorship. At the time of the purchase, Natalie, being the older sister, was in a better financial position. Therefore, Natalie contributed $300,000 and Kate contributed $100,000 to the purchase price. The property is now worth $800,000. Which of the following statements is correct? a) Natalie and Kate each own 50% of the condo. b) If Natalie were to die today, her share of the condo would transfer to her husband Brian. c) If Kate were to die today, Natalie's new basis in the property would be $400,000. d) If Natalie and Kate were to disagree on how the property was being managed, the only way they could partition their share of the property would be to find a willing buyer that would purchase both of their interests.

a. Because the property is owned JTWROS they automatically own 50% each. Answer b is incorrect because if Natalie were to die today, then her share of the condominium would transfer to Kate. Answer c is incorrect because if Kate died today, then Natalie's new basis would be $500,000 (Natalie's original $300,000 basis and Kate's step-to fair market value basis of $200,000 based on the contribution rule). Answer d is incorrect because if they disagree on how the property is being managed then either one can easily sell their share to any person. They do not need the consent of the other party.

You are opening a new financial planning practice and you would like to put together a team of experts to help your clients. Which of the following groups represents the best team to help your clients? a. Financial planner, CPA, and attorney. b. CPA, psychiatrist, and insurance salesman. c. Financial planner, attorney, and real estate agent d. Attorney, insurance salesman, and IRS agent.

a. Financial planner, CPA, and attorney. The best team for your client would include a financial planner, CPA, and attorney. A licensed insurance specialist is also a good asset to an estate planning team, but the team described in option b is not as good of a team overall as the team in option a.

Of the following, which is not a benefit of the unlimited marital deduction? a. The use of the unlimited marital deduction can shelter future appreciation of an asset from estate taxes at the death of the second-to-die spouse. b. The estate tax on property can be deferred until the death of the second-to-die spouse. c. The unlimited marital deduction can fund the applicable estate tax credit of the surviving spouse. d. The unlimited marital deduction can ensure the surviving spouse has sufficient assets to support her lifestyle.

a. Property that transfers to the second-to-die spouse is eligible for the marital deduction and, to the extent that it is not consumed, will be included in the second-to-die spouse's gross estate at the fair market value at his date of death, including any appreciation that may have occurred since the first-to-die spouse's estate. Therefore, future appreciation of an asset is not sheltered by using the unlimited marital deduction.

Kristi transferred $10,000,000 to the Kristi Family Trust. The trust is designed as an irrevocable grantor trust. Kristi retained a 5% annuity payout from the trust for the lesser of five years after the establishment of the trust or until her date of death, and she has named her only nephew, Alex, as the remainder beneficiary of the trust. Of the following statements regarding Kristi's transfer to this trust, which is true? a. Because Kristi retained the annuity interest from the trust, if she dies during the five years after the establishment of the trust, the full fair market value of the trust assets will be included in her gross estate. b. Because Kristi retained the annuity interest from the trust, she has not made a completed transfer (for gift tax purposes) to her nephew at the date she transferred $10,000,000 to the Kristi Family Trust. c. Any income within the Kristi Family Trust is taxed to the trust. d. The Kristi Family Trust is a testamentary trust because the term of the trust relates to her death.

a. The fact pattern describes a Grantor Retained Annuity Trust (GRAT) established by Kristi. If Kristi dies during the term of her annuity interest, the full fair market value of the trust assets will be included in her gross estate. Option b is false because the irrevocable transfer of the remainder interest in the trust is a completed transfer and therefore a gift. Option c is false because the income of a grantor trust is taxable to the grantor. Option d is false because the trust is an inter vivos trust (created during the grantor's life), not a testamentary trust (created in a decedent's will).

Miguel and Jane have been married for 45 years. Miguel is a citizen of Mexico, where the couple has lived for the past 25 years. Given the following list of separate property owned by Jane, and considering Jane's will leaves everything to Miguel outright, what amount would qualify for the unlimited marital deduction? A California home valued at $1,000,000. Mexican property valued at $450,000. The contents of the California home valued at $100,000. An investment account held at a New York City bank valued at $500,000 a. $0 b. $114000 c. $1,550,000 d. $1,600,000

a. $0 Because the property is transferred outright to a noncitizen spouse, it does not qualify for the unlimited marital deduction. Therefore, $0 is eligible for the unlimited marital deduction.

Gene contributed $500,000 to an irrevocable trust and did not retain any right to the trust's assets. The income beneficiary of the irrevocable trust was Gene's sister, and the remainder beneficiary of the irrevocable trust was Gene's niece. At the time of the transfer, Gene paid gift tax of $35,000. Gene died four years later, when the value of the irrevocable trust was $1,200,000. With regard to the irrevocable trust, how much is included in Gene's gross estate? a. $0 b. $35,000 c. $500,000 d. $1,200,000

a. $0 Nothing is included in Gene's gross estate. The full fair market value of the trust is excluded from Gene's gross estate because the transfer to the trust was irrevocable and Gene did not retain any right to the trust's assets. Furthermore, because more than three years have passed since the transfer, the gift tax paid will not be included in his gross estate.

Argo and his wife, who had made no previous gifts, gifted $125,000 in total present interest gifts to each of 6 grandchildren in separate accounts in the current year. They allocated their GST exemption to the accounts. How much GST tax do they each owe? a. $0 b. $156,000 after the annual exclusions c. $232,000 (40% of $582,000) d. $300,000 (40% of $750,000)

a. $0 They are allowed $5,340,000 (2014) each. They used only $582,000, net of the $28,000 per donee annual exclusion. (125,000 x 6 = 750,000 in gross gifts, less 28,000 x 6 = 168,000 which equals $582,000) They have each allocated $291,000 (1/2 of 582,000) of their $5,340,000 exemption.

Angelina contributed $25,000 in cash to a foreign charitable organization. Her AGI was $25,000. At the time of the contribution, the organization told her that her contribution was tax deductible for income tax purposes. Ignoring any income limitations, how much of the $25,000 contribution is deductible? a. $0 b. $12,500 c. $21,000 d. $25,000

a. $0 Foreign charitable organizations are not qualified charitable organizations and contributions to such organizations do not qualify for a charitable deduction. It is always the responsibility of the donee to determine the deductibility of his contribution.

Eric died on July 24, 20xx. At the time of his death, he owned 1,000 shares of Jefferson Crab stock. Given the daily trade prices for Jefferson Crab surrounding Eric's date of death, at what value will the Jefferson Crab be included in Eric's gross estate? Thursday, July 15, 20xx $101 - Price Open $107 - High $95 - Low $105 - Close Monday, July 19, 20xx $104 - Price Open $108 - High $100 - Low $103 - Close Tuesday, July 27, 20xx $103 - Price Open $105 - High $101 - Low $104 - Close Wednesday, July 28, 20xx $108 - Price Open $112 - High $108 - Low $109 - Close a. $103,290 b. $103,440 c. $103,500 d. $104,000

a. $103,290 Since the stock is not traded on the date of Eric's death, the value is determined utilizing the artificial valuation formula in the text, the average of the high and low for the 2 relevant dates, Monday and Tuesday. [($104 x 2) + ($103 x 5)]/7 = $103.29 x 1,000 shares = $103,290. Saturday and Sunday are not counted as trading days for purposes of the calculation.

Fred, the founder and CEO of WonderCo, recently passed away. At his death, Fred owned 80% of the stock of WonderCo; and the WonderCo stock was his only asset. WonderCo is a publicly traded company. Which of the following discounts would be applicable to Fred's WonderCo stock? a. Key Person Discount b. Minority Discount c. Both A and B d. Neither A nor B

a. Key Person Discount Option b is incorrect because Fred owns a controlling interest in WonderCo; therefore, a Minority Discount is not applicable. Therefore, option c is also incorrect. Option d is incorrect because answer a is correct; since Fred was both the founder and CEO of WonderCo, his stock is entitled to a Key Person Discount.

Mary's husband, Patrick, died two years ago. Patrick's will included the following three testamentary trusts: a trust for the benefit of Mary's children, but giving Mary a general power of appointment over the trust assets for the remainder of her life (GPOA Trust), a bypass trust for the benefit of Mary's children, but giving Mary a power to invade the trust assets for an ascertainable standard for the remainder of her life (Bypass Trust), and a charitable trust for the benefit of Mary's alma mater (Charitable Trust). At Mary's death, which of the trusts assets will be included in her gross estate? 1 - GPOA Trust. 2 - Bypass Trust. 3 - Charitable Trust. a. 1 only b. 1 and 2 c. 2 and 3 d. None

a. 1 only Only the GPOA Trust would be included in Mary's gross estate. Because the withdrawal right of the Bypass trust was limited to an ascertainable standard, its assets are not included in Mary's gross estate. Mary does not have an interest in the assets of the charitable trust so those assets are also not included in her gross estate.

Which one of the following transfers made this year by 85-year old Jennifer is not sooner or later subject to the Generation Skipping Transfer Tax? a. A gift of a remainder interest in a trust just established which is paying an income interest to Jennifer. The remainderman is a grandson whose parents died in an auto accident earlier this year before the inception of the trust. b. A transfer by Jennifer of $15,000 to a UTMA account established by her son for Jennifer's granddaughter. c. An irrevocable trust which pays income to Jennifer for 10 years and then pays the remainder to her grandniece who is only 25 years younger than Jennifer. d. An irrevocable trust which pays income to Jennifer's daughter for life, then distributes the remainder to the grandchild of a friend and that grandchild is currently 31 years old.

a. A gift of a remainder interest in a trust just established which is paying an income interest to Jennifer. The remainderman is a grandson whose parents died in an auto accident earlier this year before the inception of the trust. Options b, c, and d all have skip persons as transferees. Only the grandson in a is not a skip person. His parent's death moved him up a generation. The 37½ year rule does not apply to lineal descendants. The 31 year old may or may not receive assets in option d.

Elizabeth has drafted her own will using the "EZ Wills" software that she purchased on the internet. She sends it to you for a review. In your first review of the will, you look for which of the following common provisions? a. A statement of the domicile of the testator. b. A secondary clause. c. A specific bequest of property owned tenancy by the entirety. d. A disclosure clause.

a. A statement of the domicile of the testator. A statement of the domicile of the testator is a provision that is commonly found in a will. Neither a secondary clause nor a disclosure clause exist. Property owned tenancy by the entirety transfers by operation of law and is not disposed of through a will.

Which of the following statements accurately reflects the nature of buy-sell agreements? a. A stock redemption plan must have a corporation as a party to the contractual arrangement. b. A stock redemption plan increases the cost basis of surviving shareholders. c. Under a cross-purchase plan funded with life insurance, premiums paid are tax deductible to the payor. d. Proceeds of a life insurance policy owned by a surviving shareholder must be included in the gross estate of the decedent.

a. A stock redemption plan must have a corporation as a party to the contractual arrangement. The corporation must be a party to the stock redemption plan. A stock redemption plan is a stock purchase by a corporation, so the cost basis of the surviving shareholders are not affected, thus they do not receive a step up in basis. Proceeds of a policy owned by a surviving shareholder are not includible in the decedent's gross estate. Premiums are not tax deductible.

Paul would like to transfer a substantial portion of his net worth to his son, Chad. Paul believes that the assets will appreciate in value before his death, but Paul does not need any of the assets to sustain his current standard of living. However, Paul is concerned about Chad's ability to manage the assets and is afraid Chad may squander the assets. Of the following transfers, which would ensure that the assets are excluded from Paul's gross estate and could also ensure that Chad cannot squander the assets? a. An Irrevocable Trust b. An Outright Transfer c. An Installment Sale d. A Grantor Retained Annuity Trust

a. An Irrevocable Trust The assets transferred to the irrevocable trust would be excluded from Paul's gross estate, and would be subject to the management of the trustee as directed by Paul. As such, Chad would not be able to access the assets. An outright transfer (Option b) would not meet Paul's requirements because Chad would be able to access the assets immediately. Option c, an installment sale, would not meet Paul's requirements because Chad would be able to access the assets immediately, and any monies returned to Paul as installment payments would be included in Paul's gross estate. Option d does not meet Paul's requirements because if Paul dies during the term of the GRAT, the assets will be included in Paul's gross estate.

Which of the following is NOT a terminable interest? a. An ownership interest in a life insurance policy. b. A life estate in a home. c. An interest in a patent. d. An interest in property for a term equal to an individual's life.

a. An ownership interest in a life insurance policy. The ownership interest of a life insurance policy is not a terminable interest. The ownership interest does not terminate. All of the other interests listed are terminable interests. A life estate is a terminable interest because the interest in the property terminates at the individual's death. An interest in a patent is a terminable interest because a patent right terminates after a certain period of time. Option d describes a life estate, so it is also a terminable interest.

Mrs. Riley dies in 2014 leaving her entire $7.2 million estate through her will to her penniless husband, John. His estate goes to their children at his death. He has terminal cancer with a life expectancy of only 1 to 2 years. The alternative valuation date value of Mrs. Riley's entire estate is equal to $7,000,000. Select the post mortem technique John should utilize to reduce the overall estate tax liability of both estates: a. Elect Portability b. Elect to use the alternative valuation date c. Disclaim $2,000,000 and elect to use the alternative valuation date d. Do Nothing

a. Elect Portability The alternative valuation can only be used if it reduces both the gross estate (yes) and reduces the estate tax due (no, because it was all left to a spouse so no estate tax would be due in either situation). Since the new estate law permits the portability of the estate applicable exclusion between spouses, disclaiming any of the property is not necessary as Mrs. Riley's unused credit can be utilized by John in addition to his own (up to $10,680,000 in 2014).

Which of the following is NOT a requirement for a testamentary charitable bequest to qualify for the unlimited charitable deduction? a. The bequest must be contingent upon some other event occurring. b. The amount of the bequest must be determinable at the decedent's date of death. c. The fair market value of the assets must be included in the decedent's gross estate. d. The bequest must be payable to a qualifying charitable organization.

a. The bequest must be contingent upon some other event occurring. The bequest CANNOT be contingent upon some other event occurring. The bequest must be mandatory. All of the other statements are requirements for a testamentary charitable bequest to qualify for the unlimited charitable deduction.

Which of the following statement(s) concerning the choice of a stock redemption (entity agreement) versus a cross-purchase partnership buy-sell agreement funded with insurance is FALSE? a. The use of existing insurance to fund the agreement causes a transfer-for-value problem if an entity agreement is selected, but does NOT cause this problem if a cross-purchase approach is used. b. A cross-purchase should be selected if the surviving partners expect to sell their business interest during their lifetimes. c. An entity approach may solve the affordability problem if one partner is significantly older than the others. d. An entity agreement becomes more desirable as the number of partners included in the agreement increases.

a. The use of existing insurance to fund the agreement causes a transfer-for-value problem if an entity agreement is selected, but does NOT cause this problem if a cross-purchase approach is used. Transfer-for-value problems can be created if existing policies are transferred between shareholders of a corporation in a cross-purchase agreement.

Chelsea graduated from the University of Alabama. Each year, football season tickets are sold only to those who make a contribution to the university of $2,000 or more. Chelsea contributes $2,000, so that she meets the requirements to purchase season tickets, and also spends $500 on the season tickets. How much is her deductible charitable contribution for the year? a. $500 b. $1,600 c. $2,000 d. $2,500

b. $1,600 For a contribution to a university where the donor receives the right to purchase tickets to athletic events, only 80% of the contribution will be allowed as a charitable contribution. The price of the actual tickets is not deductible. $2,000 x 80% = $1,600.

Arthur transfers property valued at $250,000 to a charitable organization in return for a single life annuity based on his life value at $130,000. Arthur's adjusted basis in the transferred property was $95,000. Arthur died two years after the transaction and at the time of his death, the property had a fair market value of $325,000. At the time of the initial transfer, what was Arthur's charitable income tax deduction (ignoring any AGI limitations)? a. $75,000 b. $120,000 c. $155,000 d. $250,000

b. $120,000 The value of the property contributed less the value of the annuity received is Arthur's charitable deduction for income tax purposes. $250,000 - $130,000 = $120,000.

Mario's executor determined that the estate tax liability for Mario's estate is $600,000. However, Mario's executor forgot to file the estate tax return and filed and paid 65 days late. Calculate the penalties that Mario's estate will now have to pay. a. $81,000 b. $90,000 c. $99,000 d. $690,000

b. $90,000 The failure-to-file penalty of $90,000 (5% x $600,000 x 3 months) is reduced by the failure-to-pay penalty of $9,000 (0.5% x $600,000 x 3 months), creating an adjusted failure-to-file penalty of $81,000. Adding the failure-to-pay penalty of $9,000 to the adjusted failure-to-file penalty creates a total penalty of $90,000. Option d is incorrect because the question asks only for penalties. $690,000 is penalties plus tax.

Which of the following are characteristics of a qualified disclaimer? 1. It may not direct the bequest to another person selected by the disclaimant. It must be received by the executor of the estate within 9 months of the death of the decedent. 2. It must be written and irrevocable. 3. The disclaimant may disclaim a part of an asset. a. 1 and 2 b. 1, 2 and 3 c. 1, 3 and 4 d. 1, 2, 3 and 4

b. 1, 2 and 3 A qualified disclaimer must be written, irrevocable and received by the executor of the estate within 9 months. It must not direct the asset and can be for any interest partial or full.

Some reasons to use life insurance to fund business continuation agreements include which of the following: 1. It provides sufficient assets for the buyer to perform on the contract. 2. Insurance protects the company and its shareholder because the IRS cannot challenge the value of stock if provided for in a Shareholders Agreement (SHA). 3. The insurance gives the agreement efficacy. No money . . . No deal. 4. The insurance strengthens the commitment of the buyer when it must follow through on the agreement. a. 1, 2 and 3 b. 1, 3 and 4 c. 2 and 4 d. 4 only

b. 1, 3 and 4 The IRS may challenge the valuation of stock in business continuation agreements, with or without insurance; the IRS is not bound by any contractual agreement between the company and its shareholders.

In 1999, Price funded a bypass trust with $675,000, the applicable estate tax credit equivalency amount at that time. At Price's death in 2014, his will included a testamentary bypass trust and a residual bequest to his U.S. citizen wife. If Price's net worth at his death was $5,340,000, how much will be transferred to the bypass trust to maximize its benefits? a. $1,325,000 b. $2,825,000 c. $4,665,000 d. $5,340,000

c. $4,665,000 Price's executor would fund the testamentary bypass trust with the difference between the applicable estate tax credit equivalency at Price's death (2014 - $5,340,000) and the funding amount of the inter vivos bypass trust ($675,000). In this case, the amount would be $4,665,000 ($5,340,000 - $675,000).

Jack and his wife, Carol, were in an auto accident. Carol died three weeks before Jack did. His gross estate was $6.2 million. One of the major assets in his estate was closely held stock in an equipment leasing firm (C corporation) with which rapidly appreciating equipment was purchased. His estate had unsecured debts of $400,000 and administrative expenses of $75,000. His will allocates his estate to his children in equal shares. Which post mortem planning techniques might benefit Jack's estate? 1. The alternative valuation date. 2. A Section 303 stock redemption. 3. The QTIP election. 4. Special use valuation. 5. Installment payment of estate taxes a. 3 only b. 2 and 5 c. 4 and 5 d. 1, 3 and 4

b. 2 and 5 Due to rapid increase in asset value, statement 1 would likely provide a higher estate value and therefore the alternate valuation date is not likely useful. Statement 3 - QTIP is not an issue as Carol and Jack both die, and she is not his heir; therefore, there is no use for a QTIP. Finally, special use valuation pertains to real property used in a trade or business.

Robbie transferred $100,000 to an irrevocable trust for the benefit of his minor child, Dominic. The transfer was eligible for the annual exclusion. The trust permits the trustee to accumulate trust income within the trust, and only make distributions to Dominic based upon an ascertainable standard until Dominic is 21 years old. When Dominic attains the age of 21, the trust must terminate and the trust assets must be distributed to Dominic. Which type of trust has Robbie created? a. 2503(b) Trust b. 2503(c) Trust c. Totten Trust d. Intentionally Defective Grantor Trust (IDGT)

b. 2503(c) Trust A 2503(c) trust allows income to be accumulated within the trust until the minor beneficiary attains the age of majority and the transfer of property to the trust qualifies for the annual exclusion. A 2503(b) trust requires the trustee to make annual income distributions to the minor beneficiary. A Totten Trust is a bank account which includes a payable on death clause. An IDGT is a grantor trust which requires the grantor to pay the income tax on the income of the trust.

Bobby, a single man, owned a building with a fair market value of $2,000,000. Bobby's adjusted basis in the building was $1,000,000. In 2014 Bobby agreed to sell the building to his adult son, Robby for $1,300,000. What is the amount of Bobby's taxable gift? a. Bobby has made a taxable gift of $300,000. b. Bobby has made a taxable gift of $686,000. c. Bobby has made a taxable gift of $2,000,000. d. Bobby has not made a taxable gift.

b. Bobby has made a taxable gift of $686,000. The discount of $700,000 ($2,000,000 - $1,300,000) is treated as a gift eligible for the annual exclusion, thus creating a taxable gift of $686,000 for 2014.

Hazel, a widow, died. She had made no previous lifetime taxable gifts and she died with a gross estate of $5,250,000, consisting solely of a diversified portfolio of publicly traded, income-producing stocks. Her debts were $75,000 and estate administrative expenses amounted to $50,000. Which of the following post-mortem techniques should Hazel's executor consider electing? a. The alternate valuation date. b. Deduct estate administrative expenses on the estate's fiduciary income tax return. c. Pay estate taxes under IRC Section 6166. d. Use a Section 303 stock redemption.

b. Deduct estate administrative expenses on the estate's fiduciary income tax return. The alternative valuation is not beneficial because there is no estate liability. No estate tax is due therefore no installment payment is needed and 6166 does not apply. The estate is not a closely held business (C corporation) so Section 303 redemption does not apply.

Diana's will leaves all of her property to her husband, George. If he does not survive her by more than eight months, the property will transfer to Diana's only daughter. Diana dies on May 1 and George dies on the following December 1. Of the following statements, which is correct? a. Diana's property will transfer to her daughter and the property will be eligible for the unlimited marital deduction in Diana's estate. b. Diana's property will transfer to her daughter and the property will not be eligible for the unlimited marital deduction in Diana's estate. c. Diana's property will transfer to George and the property will be eligible for the unlimited marital deduction in Diana's estate. d. Diana's property will transfer to George and the property will not be eligible for the unlimited marital deduction in Diana's estate.

b. Diana's property will transfer to her daughter and the property will not be eligible for the unlimited marital deduction in Diana's estate. Diana's property will not transfer to George because he failed to survive her for at least eight months. Therefore, both answer c and answer d are incorrect. Option a is incorrect because the property that transfers to Diana's daughter will not be eligible for the unlimited marital deduction in Diana's estate. For transfers to a surviving spouse to qualify for the unlimited marital deduction, the survival period in the survivorship clause cannot exceed six months. Due to the length of the survivorship clause, the property would not have qualified for the unlimited marital deduction even if George survived Diana by more than eight months.

The estate planning process is fairly simple and can generally be completed by a financial planner without any assistance from a licensed attorney or CPA. a. True b. False

b. False Estate planning is a complex process that usually requires the skills of a licensed attorney and a CPA, as well as a financial planner.

Only individuals who currently have assets in excess of the applicable estate tax credit equivalency amount need estate planning. a. True b. False

b. False Everyone needs estate planning in order to minimize taxes, ensure that their loved ones receive their property, and to provide for their loved ones financially after their death.

Which of the following apply to Section 303 redemption? a. The closely held interest must meet the 25% rule. b. Qualifying redemption amounts are limited by the payment of death taxes and estate administration taxes and costs. c. A publicly traded stock will usually qualify for a 303 redemption if the interest is a minority interest. d. The stock redeemed must be common stock.

b. Qualifying redemption amounts are limited by the payment of death taxes and estate administration taxes and costs. The rule is 35% of the gross estate. The stock must be closely-held and it can be either common or preferred.

Big Mike, a very generous man, has given his granddaughter, Jordan, a gift equal to $5.34 million last year and paid any relevant taxes. He now wants to give his grandson, Colin, a gift of $5.34 million plus the annual exclusion of $14,000 on his birthday and wants to know what his total outflow will be for the gifts and any other related taxes. The amount of his cash flows related to the gift to Colin is? a. $5.354 million b. $7.476 million c. $9.626 million d. $10.4804 million

d. $10.4804 million Total guess??? Not sure why this is though! Oooops

Which of the following statements is false? a. The unlimited marital deduction is a deduction from a decedent's adjusted gross estate to arrive at the decedent's taxable estate. The unlimited marital deduction is limited to the value of the assets included in the decedent's gross estate which are transferred to the decedent's surviving spouse. b. The credit for tax paid on prior transfers was repealed in 2005. At that time, the credit became a deduction. c. If the sum of a decedent's gross estate and lifetime adjusted taxable gifts is less than the applicable estate tax credit equivalency amount for the year of the decedent's death, the executor of the decedent's estate does not have to file an estate tax return. d. Jesse gave his mom property valued at $100,000 six months before her death. Jesse's adjusted basis in the property was $45,000. Jesse was the sole heir of his mother's estate, and the same property was distributed from his mother's estate to him. At his mom's date of death, the property had a fair market of $105,000. Jesse's adjusted basis in this property is $45,000.

b. The credit for tax paid on prior transfers was repealed in 2005. At that time, the credit became a deduction. The credit for tax paid on prior transfers was NOT repealed in 2005. The state death tax credit is repealed in 2005 and is replaced with a deduction. All of the other statements are true statements.

Ralphie, a real estate mogul, dies owning a great deal of real property. Which of the following would be included in Ralphie's probate estate? a) A building owned fee simple by Ralphie's wife. Ralphie and his wife do not live in a community property state. b) A vacant lot owned joint tenancy with rights of survivorship by Ralphie and his brother. c) A beach house owned tenancy in common by Ralphie and his mother. d) An office building owned tenancy by the entirety by Ralphie and his wife.

c) A beach house owned tenancy in common by Ralphie and his mother. Option a is incorrect because the property of Ralphie's wife would not be included in his probate estate. Option b is incorrect because property owned JTWROS passes outside of probate. Option d is incorrect because property owned tenancy by the entirety passes outside of probate.

Which of the following accurately describes a QTIP Trust. a. A QTIP is sometimes called a "B" or "Q" Trust. b. Trust income must be paid to the spouse or other designated beneficiary at least annually. c. The trust assets will be included in the gross estate of the surviving spouse. d. The surviving spouse designates the remainder beneficiaries of the QTIP.

c. Option a is incorrect because a QTIP is not the same as a "B" trust. Option b is incorrect because the income of the trust must be paid to the spouse, not to any other beneficiary. Option d is incorrect because the surviving spouse does not choose the remainder beneficiaries of the QTIP.

Of the following statements, which is false? a. The availability of the unlimited marital deduction merely postpones the potential estate tax due. b. Property that is not included in the decedent's gross estate cannot qualify for the unlimited marital deduction. c. The death benefit of a life insurance policy included in a decedent's gross estate is not eligible for the unlimited marital deduction, even if the surviving spouse is the listed beneficiary and receives the proceeds. d. An individual can use the unlimited marital deduction during life to fund the surviving spouse's applicable estate tax credit. The best property to transfer in this method is the property that is expected to appreciate in value after the transfer to the surviving spouse.

c. Option c is a false statement. If the death benefit of a life insurance policy is included in a decedent's gross estate, and the surviving spouse is the listed beneficiary and receives the proceeds, the value of the death benefit will be eligible for the unlimited marital deduction. All of the other options are true statements.

Carolyn made the following transfers during her life: The transfer of her home to an irrevocable trust for the benefit of her four children on January 1, 2014. Carolyn retained the right to live in the home for the remainder of her life. The fair market value of the home at the date of the transfer to the trust was $1,000,000. The fair market value of the home at Carolyn's date of death was $1,200,000. A transfer of $44,000 to an irrevocable trust for the benefit of her four children on January 2, 2009. Carolyn retained the right to a 4% annuity payment from the trust for the years 2009 and 2010. At Carolyn's date of death, the trust had a value of $62,000. If Carolyn died on July 13, 2014, with regard to the above transfers, how much is included in Carolyn's gross estate? a. $0 b. $1,044,000 c. $1,200,000 d. $1,262,000

c. $1,200,000 Carolyn's gross estate would include the fair market value of the home at her date of death, but not the value of the trust listed in #2. The transfer listed as #1 would be included in Carolyn's gross estate because Carolyn retained an interest in the home that terminated at her death. Therefore, the full fair market value of the transferred property would be included in the transferor's gross estate at the time of the transferor's death. No amount related to the transfer listed as #2 would be included in Carolyn's gross estate because the annuity interest terminated before Carolyn's death.

Lisa made the following transfers during 2014: - $17,000 to her grandson for his law school tuition. - $1,000 to her neighbor to help him pay a hospital bill. - A transfer of property valued at $100,000 to a GRAT. Lisa retained an annuity valued at $40,000 and her daughter is the remainder beneficiary. What is the total amount of Lisa's taxable gifts for 2014? a. $48,000 b. $60,000 c. $63,000 d. $65,000

c. $63,000 Lisa's transfers to her grandson and neighbor are not qualified transfers because the payments were not made directly to the educational or medical institution. The transfers are eligible for the annual exclusion, though. As such, the taxable amount of each is $3,000 ($17,000 - $14,000) and $0, respectively. The transfer of the remainder interest in the GRAT to Lisa's daughter is valued at $60,000 ($100,000 - $40,000). Since it is a gift of a future interest, the transfer is not eligible for the annual exclusion. Accordingly, Lisa's total taxable gifts are $63,000 ($3,000 + $0 + $60,000).

When Ronnie died seven months ago he left his prize art collection to his daughter Kate. Ronnie had a fantastic eye for selecting artwork by unknown painters, buying the painting cheap, and then selling them for a high profit once the painter was recognized by the general public. Three months before his death, Ronnie purchased an enchanting oil painting of a beautiful women that Ronnie claimed would be "as famous as the Mona Lisa" for $4,000. Kate has been exhibiting the painting since her father's death and a local art collector offered her $100,000 for the painting. Kate is extremely excited because the painting was only valued at $15,000 when her father died. If Kate sold the painting today, what would her taxable gain be for income tax purposes. a. $85,000 short term capital gain. c. $85,000 long term capital gain. d. $96,000 short term capital gain. e. $96,000 long term capital gain.

c. $85,000 long term capital gain. Kate's basis in the property is equal to the date of death value. The holding period for inheritances is long term regardless of how long the decedent or the legatee held the property. Thus her gain is $100,000 (sale price) - $15,000 (Kate's basis in the property). Her holding period is a long term capital gain.

A client asks you to explain the statement, "Life insurance proceeds are tax-free." You answer that the general rule(s), subject to some exceptions, is/are that death benefits received from a life insurance policy due to the death of the insured are income tax free to the beneficiary, but which of the following are also correct: 1. The proceeds are subject to estate taxes in the estate of the insured if the insured is the owner. 2. The proceeds may be subject to income taxes if the policy was sold to a third party. 3. The proceeds are not subject to income tax, even if sold to a third party if the contract is a modified endowment contract a. 1 only b. 2 and 3 c. 1 and 2 d. 1, 2 and 3

c. 1 and 2 Death benefits from a modified endowment contract (MEC) are still income tax free unless sold. Statement 2 subjects the policy proceeds to income tax if the policy was transferred to a transferee who took under a transfer for value rule.

Federal estate and gift taxes are determined by the fair market value of the property transferred. Which of the following statements are true? 1. Asset values are based upon the fair market value on the date of death or six months after the date of death for the gross estate. 2. Taxes are progressively higher as more assets are transferred during life. 3. Value is determined on the date of the transfer of the assets for lifetime transfers. 4. Special use valuation is always available for special use property. a. 1 only b. 1 and 3 c. 1, 2 and 3 d. 1, 2, 3 and 4

c. 1, 2 and 3 The value of the assets transferred may use an alternative valuation date. Special use valuation is only available if certain qualifications are met (see 2032 (a)).

Which of the following describes joint and survivorship life insurance? 1. It is generally not includible in any insured's gross estate, if owned in an ILIT. 2. It can provide liquidity to pay estate taxes at the death of the second insured. 3. It pays a partial benefit at the death of the first to die (administrative and estate taxes) with the remainder paid in full at the second death. 4. Premiums are usually less expensive than for individual policies on each of the two insureds for the same face amount. a. 1 and 2 b. 3 and 4 c. 1, 2 and 4 d. 1, 2, 3 and 4

c. 1, 2 and 4 Survivorship life pays the entire death benefit at the second death and is generally not included in the insured's gross estate if owned in an ILIT.

Making arrangements to deal with the possibility of physical or mental incapacity is an important area of estate planning. Which of the following arrangements may be used to deal with such unexpected incapacity? A springing durable power of attorney. A revocable living trust. Fee simple titling. A living will. a. 1 only b. 2 and 4 c. 1, 2, and 4 d. 1, 2, 3, and 4

c. 1, 2, and 4 Fee simple ownership is not an arrangement that helps to deal with unexpected incapacity. All of the other arrangements are methods of dealing with unexpected incapacity.

Which statement(s) is/are true for Generation Skipping Transfer Tax (GSTT)? 1. Applies to transfers to persons who are two generations or more lower than the transferor. 2. There are no exceptions. 3. There is a $5,340,000 lifetime exemption in 2014 for GSTT. 4. Transfers qualifying for gift tax annual exclusion are also excluded from GSTT Choose the answer(s) which is/are most correct: a. 1 and 4 b. 2 and 3 c. 1, 3, and 4 d. 4 only

c. 1, 3 and 4 There are exceptions to the GSTT including the predeceased parent rule. Gifts qualifying for the annual gift tax exclusion are excluded for GSTT. The lifetime exemption for GSTT is $5,340,000 (2014).

Which of the following statements regarding SCINs is correct? a. If the seller outlives the SCIN term, the buyer continues to pay the SCIN payment until the seller's death. b. The payments received by the seller under a SCIN are treated as interest income. c. A SCIN can give the seller a collateral interest in the property sold. d. If the seller dies before the end of the SCIN term, the seller is deemed to have made a taxable gift to the buyer equal to the difference between the payments made and the total principal payments due on the SCIN.

c. A SCIN can give the seller a collateral interest in the property sold. Option a is incorrect because the buyer of a SCIN only makes payments until the earlier of (1) the seller's death or (2) the term set forth in the SCIN. Option b is incorrect because each payments received by the seller consists of (1) interest income, (2) capital gain, and (3) return of adjusted basis. Option d is an incorrect statement. If the seller dies before the end of the term, the difference between the seller / decedent's adjusted basis and the face value of the note is deemed a transfer of the estate, and must be included as income on the estate's tax return.

Which of the following qualifies for the unlimited marital deduction? a. An outright bequest to resident alien spouse. b. Property passing to a noncitizen spouse in a QTIP. c. An outright bequest to a resident spouse who, prior to the decedent's death was a noncitizen, but who after the decedent's death and before the estate tax return was filed, became a U.S. citizen. d. An income beneficiary of a CRUT who is a nonresident alien spouse.

c. An outright bequest to a resident spouse who, prior to the decedent's death was a noncitizen, but who after the decedent's death and before the estate tax return was filed, became a U.S. citizen. Of the options, only an outright bequest to a resident alien spouse who becomes a U.S. citizen before the estate return is filed qualifies for the unlimited marital deduction.

Donna has AGI of $100,000. Donna owns a rare antique in which she has an adjusted basis of $200,000. The antique is currently worth $2,000,000. Assuming that Donna's AGI will remain at $100,000 for the next six years, which of the following would you recommend to her if she donates the antique to a museum this year? a. Donna should deduct the entire fair market value of the antique this year. b. Donna should deduct $30,000 this year and every year for the next five years. c. Donna should deduct $50,000 this year. d. Donna should deduct $200,000 this year.

c. Donna should deduct $50,000 this year. Option c is correct because, given Donna's AGI, she will obtain the maximum tax benefit by electing to deduct the adjusted basis of the antique, subject to a ceiling of 50% of her AGI. Electing to deduct the adjusted basis produces a deduction of $50,000 per year for four years, for a total charitable deduction of $200,000 (equals basis). Option b is wrong because if Donna elects to deduct the fair market value of the antique, and is thus limited to 30% of her AGI, her deduction will be $30,000 per year for six years, or a total of $180,000, which is less advantageous than option c. Donna may not take the deductions described in option a and option d.

Which of the following is true regarding a Grantor Retained Annuity Trust (GRAT)? a. At the end of the GRAT term, a taxable gift occurs. b. If the grantor dies during the trust term, a pro rata portion of the trust assets will be included in the grantor's estate. c. Interest and dividends earned by assets in a GRAT are taxed to the grantor. d. If the grantor survives the trust term, all of the trust assets will be included in the grantor's estate.

c. Interest and dividends earned by assets in a GRAT are taxed to the grantor. Option a is incorrect because a taxable gift occurs when the GRAT is established, not when the GRAT term ends. Option b is incorrect because if the grantor dies during the trust term, all of the trust assets are included in his gross estate. Option d is incorrect because if the grantor survives the trust term, none of the trust assets are included in his estate.

Nellie recently executed a power of attorney giving Jessie the power to perform certain tasks. Which of the following powers given to Jessie would cause the power to be deemed a general power of appointment? a. Nellie gave Jessie the power to use Nellie's money to pay Nellie's creditors. b. Nellie gave Jessie the power to sell and buy property on Nellie's behalf. c. Nellie gave Jessie the power to use Nellie's money to pay Jessie's creditors. d. Nellie gave Jessie the power to make gifts to Nellie's heirs and charities.

c. Nellie gave Jessie the power to use Nellie's money to pay Jessie's creditors. Giving Jessie the power to pay his own creditors creates a general power of appointment over the assets. The other powers do not benefit Jessie and thus do not create a general power of appointment.

You are a CFP and although you never went to law school, you consider yourself to be very good at reviewing wills. Your client, Catherine, asks you to prepare a will for her. Should you prepare a will for Catherine? a. Yes, Catherine is your best client and you might lose her if you do not prepare the will. b. Yes, it is permissible for a CFP to prepare a legal document. c. No, preparing Catherine's will would be considered the unauthorized practice of law. d. No, you should only prepare Catherine's will if you are going to prepare her husband's will as well.

c. No, preparing Catherine's will would be considered the unauthorized practice of law. Drafting legal documents, such a wills, is an activity reserved for licensed attorneys. If you are not a licensed attorney and you prepare a legal document, you have engaged in the unauthorized practice of law.

Although he has a vast fortune, Ricky has decided not to prepare an estate plan because he believes that his surviving family members will divide up his assets appropriately. Which of the following is not a risk associated with failing to plan an estate? a. Ricky's estate could incur excessive transfer taxes. b. Ricky's favorite Corvette may not be transferred to his ex-wife, Carla. c. Ricky's insurance policy on his own life may not be paid out to the named beneficiary. d. Ricky's current wife, Lucille, may not provide for Ricky's children from a previous marriage.

c. Ricky's insurance policy on his own life may not be paid out to the named beneficiary. The proceeds of insurance policies with named beneficiaries pass outside of probate via state contract law. Ricky's failure to plan his estate will not affect his insurance policy.

Gina, age 79, recently had a stroke. Afraid that she may not live long enough to see her family enjoy her beach house, she would like to transfer it to her daughter, Taylor. Gina does not want to pay any gift tax or utilize any of her lifetime credit amount. Which of the following techniques, if used by Gina to transfer the beach house to Taylor, will not result in a taxable gift? a. GRAT b. QPRT c. SCIN d. GRUT

c. SCIN A SCIN is a note with a self-cancelling premium payment attached so that the note will cancel at the transferor's death. The GRAT, QPRT and the GRUT are irrevocable trusts and will result in a current taxable gift.

Which of the following are parties to a power of attorney? a. The principal's mother, even though she is not named as the principal's agent. b. The guardian ad litem. c. The principal, or person granting the power. d. The attorney who prepares the power of attorney.

c. The principal, or person granting the power. The parties to a power of attorney are the principal and the agent.

During the year, Edward created a trust for the benefit of his five children. The terms of the trust declare that his children can only access the trust's assets after the trust has been in existence for 20 years and the trust does not include a Crummey provision. If Edward transfers $100,000 to the trust during the year, what is his total taxable gift for the year? a. $0 b. $30,000 c. $60,000 d. $100,000

d. $100,000 Because the trust does not include a Crummey provision, the transfer to the trust is a gift of a future interest and is not qualified to be offset by the annual exclusion. Therefore, the entire transfer to the trust is subject to gift tax.

Maxwell died August 8, 2014. Of the following transfers made during his life, which is included in his gross estate? a. The transfer of a whole life insurance policy on Maxwell's life to an ILIT on September 16, 2010. b. The sale of his term insurance policy to his brother, Donald, for fair market value on August 12, 2010. c. The transfer of a whole life insurance policy on Maxwell's life (face value $150,000) valued at $20,000 to his son on September 16, 2012. d. A gift of $14,000 to Maxwell's sister on August 7, 2014. No gift tax was due on the gift.

c. The transfer of a whole life insurance policy on Maxwell's life (face value $150,000) valued at $20,000 to his son on September 16, 2012. The transfer would be included in Maxwell's gross estate because transfers of life insurance on the decedent's life within three years of the decedent's date of death are included in the decedent's gross estate. Option a is incorrect because the transfer is not included in Maxwell's gross estate because the transfer was completed more than three years prior to Maxwell's date of death. Option b is incorrect because the sale of an insurance policy for fair market value removes the asset from the gross estate. Option d is incorrect because gifts, other than life insurance, within three years of the decedent's date of death are not included in the decedent's gross estate. Gift tax paid within three years of the decedent's date of death is included in the decedent's gross estate, but in this case no gift tax was paid.

Raymond's net worth is $10,000,000, consisting entirely of his separate property. His wife's net worth is $200,000, consisting entirely of her separate property. As part of Raymond's estate plan, he would like to transfer as much to his wife as possible, while making the most of his applicable estate tax credit. He would also like to ensure that his wife has access to all of his net worth for the rest of her life. It is his wish that at his wife's death, the children will inherit whatever is left, with the least possible transfer taxes owed. Which of the following estate plans would fulfill Raymond's goals? Assume that Raymond dies in 2014, and that he has not used any of his exemption amount. a. Raymond's estate could transfer $10,000,000 to Raymond's wife as a specific bequest, and direct that his estate's executor donate Raymond's entire unused exemption to his wife. b. At Raymond's death, his estate could create an ILIT for the sole benefit of Raymond's children utilizing Raymond's remaining applicable estate tax credit equivalency. These funds could be used to purchase life insurance on Raymond's wife. Any remaining assets should be given the Raymond's wife outright. c. At Raymond's death, his estate should transfer $5,250,000 to an Irrevocable Trust for the sole benefit of Raymond's children. The remaining assets should be given outright to Raymond's wife. d. At Raymond's death, $5,340,000 should be transferred to a bypass trust for the benefit of Raymond's children. Raymond's wife should be given the power to invade the bypass trust for an ascertainable standard. The remaining assets should be given outright to Raymond's wife.

d. Only option d would give Raymond's wife access to all of his estate, and utilize his applicable estate tax credit; therefore, option d is the best option. Option a is not the best option because we do not know what the state of the law portability will be at Raymond's wife's death. And, while option a assumes that Raymond's wife's now ten million dollar estate will pass to her beneficiaries with no estate tax, she could lose Raymond's unused exemption amount by remarrying and becoming the surviving spouse of another. Moreover, by giving all of his fortune outright to his wife, it is possible that none will go to his children. Option b may maximize future estate benefits for the Raymond's children after Raymond's wife's death, but does not meet Raymond's desires to allow his wife to access all of the funds. Raymond's wife would not have access to the funds within the ILIT. Option c, like option b, does not meet Raymond's goals because Raymond's wife would not have the right to access the funds within the Irrevocable Trust.

Which of the following statements is true? a. Angela transferred her home to a QPRT in 2010. She retained the right to live in the home for 13 years, and at the end of the term, the home transfers to Angela's three children. Angela dies in 2013, when the home has a fair market value of $250,000. The value of the home is excluded from Angela's gross estate because the children are the remainder beneficiaries of the QPRT. b. Missy transfers rental property to a Family Limited Partnership (FLP) in return for a 99% limited partnership interest and a 1% general partnership interest. Missy immediately begins a gifting program by gifting a portion of the limited partnership interests to her children and grandchildren. Six years after the initial formation of the FLP, Missy continues to own the 1% general partnership interest and 45% of the limited partnership interests in the FLP. Because Missy does not own a majority (greater than 50%) of the interest in the FLP, Missy cannot control the operations of the FLP. c. Rose has been diagnosed with an illness that is expected to substantially reduce her life expectancy. Before she dies, Rose would like to transfer her extremely valuable art collection to her wealthy daughter. The art collection is displayed in Rose's home, but Rose really needs money for her living expenses for the remainder of her life. A TPPT would be the most appropriate transfer device to fulfill Rose's desires. d. Earl has four children - Kenny, Tim, Aaron, and Cathy. Earl's will directs all of his property to be divided equally among his four children, and if any child predeceases Earl, that child's heirs will inherit Earl's property per capita. Cathy died two years before Earl. Cathy had three children. At Earl's death, Kenny will receive 1/6 of Earl's estate.

d. Option d is correct because Cathy's surviving heirs become equal heirs in Earl's estate because Cathy predeceased Earl. Accordingly, each heir receives 1/6 (Kenny, Tim, Aaron, Cathy's three kids) of Earl's estate. Option a is incorrect because the value of the home transferred to the QPRT will be included in Angela's gross estate because Angela died during the term of the QPRT. Option b is incorrect because ONLY the general partner can control the operations of a limited partnership and Missy is the general partner so she CAN control the operations of the FLP. Option c is incorrect because a TPPT would not fulfill Rose's desires because the TPPT would only give Rose the right to use the art for the rest of her life, and would not provide any income.

In 2012, Lori assigned a paid-up whole life insurance policy to an Irrevocable Life Insurance Trust (ILIT) for the benefit of her three children. The ILIT contained a Crummey provision for the benefit of each child. At the time of the transfer, the whole life insurance policy was valued at $200,000, and since Lori had not made any other taxable gifts during her lifetime, she did not owe any gift tax. Lori died in 2013, and the face value of the whole life insurance policy of $2,000,000 was paid to the ILIT. Regarding this transfer, how much is included in Lori's gross estate at her death? a. $0 b. $164,000 c. $964,000 d. $2,000,000

d. $2,000,000 The death benefit of a life insurance policy transferred within three years of the decedent's date of death is included in the decedent's gross estate. In this case, Lori transferred the policy one year before her death, so the full death benefit of $2,000,000 is included in her gross estate.

Jennifer purchased her mother's home through the use of a SCIN. Under the terms of the SCIN, Jennifer was to pay her mother $22,000, plus interest and a SCIN premium, per year for 10 years. If Jennifer's mother died after six payments were made, what would be Jennifer's adjusted basis in the home? a. $0 b. $88,000 c. $132,000 d. $220,000

d. $220,000 The buyer's adjusted basis in property transferred through the use of a SCIN is the fair market value of the property on the date of the sale regardless of the number of payments made by the seller. In this case, the fair market value of the property must have been the annual principal payments times the expected term of the SCIN, or $220,000 ($22,000 x 10).

Maxine agrees to purchase Jacob's property utilizing a private annuity. Jacob's table life expectancy is ten years at the date of the agreement and the property has a fair market value of $400,000. The private annuity payment is $45,000 per year, and Maxine dies after making two payments. At Maxine's death, what amount is included in her gross estate with regards to the private annuity and the transferred property? a. $0 b. $90,000 c. $310,000 d. $400,000

d. $400,000 Maxine bought the property utilizing the private annuity. Maxine's gross estate will include the fair market value of the property purchased. The expected present value of the remaining private annuity payments will be a debt of the estate.

Steve made the following transfers during the year: $10,000 to Louisiana State University. The $10,000 contribution allows him to purchase football season tickets. Steve also bought the football season tickets at a cost of $5,000. $400 to the local public broadcast television station during the annual fund drive. In return for the $400 contribution, Steve received a mug and pen with the station's logo valued at $8. 1,000 shares of ABC stock to the United Way. At the date of the contribution, the stock had a fair market value of $50 per share. Steve's adjusted taxable basis in the stock was $10 per share and he held the stock long term. Ignoring any AGI limitations, what is Steve's maximum charitable income tax deduction for this year? a. $18,400 b. $20,392 c. $55,392 d. $58,400

d. $58,400 Ignoring any AGI limitations, Steve could deduct 80% of the $10,000 contribution, or $8,000. The $400 contribution to the public broadcast television station will not be reduced by the value of the mug and the pen because those items are considered de minimus. Without any AGI limitations, the full fair market value of the stock contribution may be deductible, $50,000 ($50 x 1,000). $8,000 + $400 + $50,000 = $58,400.

Death benefit proceeds from a life insurance policy are included in a decedent's gross estate in which of the following circumstances: 1. The decedent gave the policy to his father four years ago, but retained the right to change the name of the beneficiary. 2. The policy beneficiary is a grantor trust of the decedent but the policy is owned by a closely-held corporation. 3. The decedent gave the policy to a charity seven years ago. 4. The decedent transferred the policy to an irrevocable life insurance trust five years ago with no retained incidents of ownership. a. 1 and 4 b. 2 and 3 c. 3 and 4 d. 1 and 2

d. 1 and 2 Statement 1 is included because the decedent retained an ownership right and statement 4 was transferred more than 3 years ago.

Which of the following accurately reflects the use of split-dollar life insurance in a business setting? 1. It can be a fringe benefit to an employee. 2. The insurance premiums are usually split between the employer and the employee (insured). 3. It may be used to fund a buy-sell stock redemption agreement. a. 1 only b. 1 and 2 c. 2 and 3 d. 1, 2 and 3

d. 1, 2 and 3 All these statements are correct. Split dollar life insurance is an arrangement where an employee and employer generally share the premium cost and cash value for death benefit of a life insurance policy covering the life of the employee.

Which of the following constitute incidences of ownership in an insurance policy: 1. The right to name or change the name of the beneficiary. 2. The right to surrender the policy. 3. The right to assign the policy. 4. The right to borrow cash from the policy. a. 3 and 4 b. 2 and 3 c. 1, 2 and 4 d. 1, 2, 3 and 4

d. 1, 2, 3 and 4 All of these rights are incidences of ownership.

Use of an Irrevocable Life Insurance Trust can accomplish which of the following? 1. Create a vehicle to avoid Generation Skipping Transfer Tax. 2. Make proceeds available to the surviving spouse. 3. Ensure that proceeds will be excluded from the probate of both spouses. 4. Shelters cash contributed for premiums from taxation up to the annual exclusion amount. a. 1 and 2 b. 2, 3 and 4 c. 1, 2 and 3 d. 1, 2, 3 and 4

d. 1, 2, 3 and 4 An ILIT will accomplish all of the items listed in this question.

The Generation Skipping Transfer Tax (GSTT) has all the following characteristics, except: a. GST gifts to direct skips qualifying for the annual exclusion are not subject to the tax. b. Assets transferred to a trust that has a grandchild as the sole beneficiary may be subject to both gift and generation skipping transfer tax. c. If all the children of a trust are grandchildren (whose parents are living) of the grantor then the trust is subject to GSTT. d. A "skip person" is a person who is one or more generations younger than the transferor.

d. A "skip person" is a person who is one or more generations younger than the transferor. Options a, b, and c are true but in the case of option d, a grandchild whose parent has died has moved up a generation with regard to skip-person considerations. A skip beneficiary is generally a person who is two or more generations younger than grantor.

Colin would like to use his recent inheritance of $200,000 to establish a charitable remainder trust. Colin would like to have the flexibility to make additional contributions to the charitable remainder trust in the future. Which of the following would you recommend for Colin? a. A Charitable Remainder Annuity Trust. b. A Charitable Gift Annuity. c. A Charitable Lead Unitrust. d. A Charitable Remainder Unitrust.

d. A Charitable Remainder Unitrust. Option a is incorrect because additional contributions may not be made to a CRAT. Option c is incorrect because a CLUT is not a charitable remainder trust. Option b is incorrect because each donation is a separate annuity and the annuity it not a remainder trust.

The best life insurance policy for the payment of federal estate taxes for a 55-year-old couple with illiquid assets is: a. An individual whole-life policy on each spouse on a cross-ownership basis. b. A joint first-to-die life insurance policy owned jointly. c. A joint last-to-die life insurance policy owned by the spouse with the larger estate. d. A joint and last-to-die life insurance policy owned by an irrevocable life insurance trust.

d. A joint and last-to-die life insurance policy owned by an irrevocable life insurance trust. A second-to-die life policy provides insurance for a lower cost than insuring each spouse individually. Due to the unlimited marital deduction, there is no need for liquidity until the death of the second spouse. The ILIT keeps the insurance proceeds from being included in the gross estate of either spouse as long as neither has any incidence of ownership in the trust or policy.

Your client, Albert, is 68-years old. He is interested in establishing a trust with a value of $6,000,000 for his family. He is aware of the Generation Skipping Transfer Tax, and he has asked you for your advice as to which of the following would be considered a skip person. Which of the following is a skip person? a. Albert's son Patrick, who is age 17. b. Albert's grandson Connor, age 14, whose mother (Albert's daughter) died in an auto accident this year. c. Albert's mother Thelma. d. A trust that Albert had established 3 years ago for Albert's favorite employee, Sam, who has just turned 20.

d. A trust that Albert had established 3 years ago for Albert's favorite employee, Sam, who has just turned 20. Due to the age difference of more than 37½ years and the non-related party status, the trust for Sam is a skip person. The reason Patrick is not a skip person is because he is a first generation descendant. Connor is not a skip person because his mother's death moves him up a generation (predeceased parent rule).

Which of the following are included in the gross estate: a. Proceeds from a life insurance policy owned by the decedent insured that was assigned to an ILIT two years before death of the insured. b. A secular trust where the only income beneficiary was the decedent's spouse. c. Property where the decedent had a reversionary interest of less than 1% of the value. d. Gift taxes paid two years prior to the decedent's date of death for gifts made four years earlier.

d. Gift taxes paid two years prior to the decedent's date of death for gifts made four years earlier. Incidence of ownership of life insurance policies assigned within three years of death are includible in the decedent's estate, as are CRATs and CRUTs. Any amount of gift tax subject to the gross up rule is includible in the taxable estate but must be for gifts made within three years of death.

Marie is the founder and sole owner of Purple Cakes Bakery. Allen has offered to buy her business for a price Marie considers reasonable, but Allen does not have all of the funds necessary to pay for the business at the current time. Marie is in good health, her true life expectancy is much greater than the IRS life expectancy factor, and she wants to accept Allen's offer. Allen is not related to Marie and has good credit. Given these facts, which transfer method should be used to transfer the business to Allen? a. Grantor Retained Annuity Trust. b. Self-Cancelling Installment Note. c. Private Annuity. d. Installment Sale.

d. Installment Sale Marie would sell the business to Allen utilizing an installment sale and would charge a reasonable rate of interest. Because Allen would not have to pay the full sale price at the date of the transfer, he would not need to have all of the funds necessary at that time. Because Allen is not related to Marie, she would not have any reason to enter into a GRAT, SCIN, or Private Annuity, which may inequitably benefit Allen. The best situation would be for Marie to sell the business to Allen in an outright cash sale, but that is not an option in this problem.

XYZ Corporation is a closely held corporation. Martin McFly, along with the three other owners, set up a stock redemption agreement requiring the corporation to buy all shares of a deceased or disabled shareholder. The plan is funded by entity life insurance policies on each shareholder. Premiums are paid by the corporation. The agreement states that the share price of any budget will be established by an independent, competent third party appraiser. What are the tax implications of this plan? 1. A deceased shareholder's gross estate will be increased by the amount of the life insurance. 2 There is no step-up in basis for decedent's family on the shares of stock covered by the plan. 3 The corporation will owe income tax on the difference between the cash value of the policy and the death benefit amount a. 1, 2 and 3 b. 1 and 3 c. 2 only d. None of the above

d. None of the above The deceased shareholder's estate will not increase due to the life insurance, as the deceased shareholder does not own the insurance policy and already has the value of his business interest in his gross estate. There is a step-up in basis because the descendant died and the shares are "purchased" by the corporation. The corporation is "owed" the premiums by the individual at death and does not pay tax.

Reese donated $100 to her church and $300 to the United Way. Which of the following is true with regard to her contribution to the charitable organizations? a. Reese must file IRS Form 8283. b. Both her church and the United Way are required to send a confirmation of the contribution to Reese. c. Only her church is required to send a confirmation of the contribution to Reese. d. Only the United Way is required to send a confirmation of the contribution to Reese.

d. Only the United Way is required to send a confirmation of the contribution to Reese. Option a is incorrect because IRS Form 8283 must be filed whenever the aggregate total of all non-cash contributions exceeds $500. Options b and c are incorrect because contemporaneous written acknowledgement by the donee organization is only required when an individual contributes cash or property valued at $250 or more. Therefore, Reese's church is not required to send a confirmation of Reese's donation.

Which of the following is NOT a reason that the death benefit of a life insurance policy would be included in a decedent's gross estate? a. The beneficiary of the policy is the estate of the decedent b. The decedent transferred the ownership of the policy to his daughter 6 years before his death, but retained the right to change the beneficiary of the policy. c. The decedent transferred the ownership of the policy to his son six months before his death. d. The decedent transferred the ownership of policy to his partner four years ago.

d. The decedent transferred the ownership of policy to his partner four years ago.

Which of the following is not a reason that the death benefit of a life insurance policy would be included in a decedent's gross estate? a. The beneficiary of the policy is the estate of the decedent. b. The decedent transferred the ownership of the policy to his daughter six years before his death, but retained the right to change the beneficiary of the policy. c. The decedent transferred the ownership of the policy to his son six months before his death. d. The decedent transferred the ownership of the policy to his partner four years ago.

d. The decedent transferred the ownership of the policy to his partner four years ago. Option a is incorrect because the proceeds of the policy would be included in the estate if the proceeds are payable to the estate. Option b is incorrect because the decedent is considered to have an incident of ownership in the policy if he retains the right to change the beneficiary of the policy. Option c is incorrect; under IRC Section 2035, the proceeds of a policy transferred within three years of death are included in the gross estate of the transferor.

Which of the following is not a reason that the proceeds of a life insurance policy would be included in a decedent's gross estate a. The proceeds of the policy are payable to the estate. b. The decedent transferred the ownership of the policy to his daughter six years before his death, but retained the right to change the beneficiary of the policy. c. The decedent transferred the ownership of the policy to his son six months before his death. d. The decedent transferred the ownership of the policy to his wife four years ago.

d. The decedent transferred the ownership of the policy to his wife four years ago. Option a is incorrect because the proceeds of the policy would be included in the estate if the proceeds are payable to the estate. Option b is incorrect because the decedent is considered to have an incident of ownership in the policy if he retains the right to change the beneficiary of the policy. Option c is incorrect; under IRC section 2035, the proceeds of a policy transferred within three years of death are included in the gross estate of the transferor.

Which of the following is not a requirement of the unlimited marital deduction? a. In order to claim a marital deduction, the decedent must have been married as of the date of his death. b. The surviving spouse must be a U.S. citizen. c. The surviving spouse must be a U.S. citizen. d. The gross value of qualifying property left to the surviving spouse is included in the marital deduction.

d. The gross value of qualifying property left to the surviving spouse is included in the marital deduction Options a, b, and c are all requirements of the unlimited marital deduction. Option d is incorrect because only the net value, not the gross value, of qualifying property left to the surviving spouse is included in the marital deduction. The term "net value" for marital deduction purposes equals the gross value of the qualifying property left to the surviving spouse less any taxes, debts, or estate administration expenses payable out of the spousal interest.

Which of the following rights will not cause an insurance policy to be included in the gross estate of the owner/insured if retained within the three years prior to the death of the owner/insured assuming the policy was in an ILIT? a. The right to pay the annual premium directly to the insurer. b. The right to assign the policy, but only to a qualified charity. c. The right to surrender the policy, but only in case of terminal illness. d. The right to change the name of a charitable beneficiary to another charitable beneficiary.

d. The right to change the name of a charitable beneficiary to another charitable beneficiary. Any incidence of ownership (answers a, b or c) constitute incidence of ownership and would cause the policy to be included in the gross estate. The right to change a charitable beneficiary to another charitable beneficiary is not an incidence of ownership because the first charitable beneficiary may no longer exist.

Tracey is a financial planner who received his CFP designation. Tracey does not have any other designations or licenses. Although Tracey's expertise is investment planning, he is anxious to expand his client base and is willing to assist clients with any area of financial planning. Over the last month Tracey engaged in the following activities with Troy, a new client. a. During the initial meeting, Tracey collected personal data about Troy including the estate planning documents Troy had previously executed. b. During the second meeting, Tracey recommended the use of a trust to fulfill some of Troy's estate planning goals. c. Troy called Tracey one afternoon and asked if Tracey could explain the probate process to him, which Tracey promptly did. d. Tracey downloaded a copy of a generic will from the internet, filled in Troy's information and gave the document to Troy to be executed. Of the activities above, which would be considered the unauthorized practice of law?

d. Tracey downloaded a copy of a generic will from the internet, filled in Troy's information and gave the document to Troy to be executed. Only activity d) would be considered the unauthorized practice of law. The drafting of legal documents is reserved for attorneys. Inquiring about estate planning documents should be completed by all practitioners. Recommending appropriate estate planning devises, such as trusts, can be done by financial planners. Explaining the probate process to a client would not be the unauthorized practice of law; the line would be crossed if Tracey gave legal advice regarding the probate process.


Related study sets

Introduction to Comparison/Contrast Essays

View Set

Area: Finding Area on a Coordinate Plane

View Set

Mill Ch. 2 (What Utilitarianism is)

View Set

Accounting 201 - Chapter 10 Quiz

View Set

Informatics and Professional Identity

View Set

Business Finance Chapter 7, Business Finance Chapter 8, Business Finance Chapter 9, Business Finance Final Exam review, Business Finance Exam 3 Final Exam, Finance Test 3, Business Finance Ch. 6, Business Finance Chapter 9, Business Finance 3, Busine...

View Set